No one in the French department to which Professor Alban belongs is allowed to teach more than one introductory level...

Batman on January 11, 2014

Need your help

Please explain why the rest of choice does not match with the structure of the passage. Thanks.

Replies
Create a free account to read and take part in forum discussions.

Already have an account? log in

Naz on January 18, 2014

Let's diagram!

"No one in the French department to which Professor Alban belongs is allowed to teach more than one introductory level class in any one term"

ATM1L = allowed to teach more than one introductory level class in any one term

P1: FDPA ==> not ATM1IL
ATM1L ==> not FDPA

"Moreover, the only language classes being taught next term are advanced ones."

LCTNT = language class taught next term

P2: LCTNT ==> A
not A ==> not LCTNT

"So it is untrue that both of the French classes Professor Alban will be teaching next term will be introductory level classes."

Let's see what the method of reasoning is here. We know that this is a valid argument. However, it's important to note that each premise given to us by itself is sufficient to conclude that it is untrue that both of the French classes Professor Alban will be teaching next term will be introductory level classes. P1 and P2 lead us to the deduction that at least one of the French classes is not going to be introductory because no one in the French department to which Professor Alban is allowed to teach more than one introductory level class in any one term and if a language class is to be taught next term, they are advanced. So, our correct answer will also have two premises that are each sufficient to derive the conclusion.

(A) is not correct because it does not parallel the reasoning displayed in the argument. Let's diagram:

"If a building is occupied by May the new tax rates apply to it"

P1: BOM ==> NTRA
not NTRA ==> not BOM

"The Morrison Building will be fully occupied by May"

P2: BOM

"The Morrison Building will be taxed according to the new rates."

C: NTRA

Here we have a valid positive argument invoking P1: BOM ==> NTRA. As you can see, we need both premises for the conclusion. Meaning, answer choice (A) does not parallel the method of reasoning in the stimulus.

(B) is not correct because it does not parallel the reasoning displayed in the argument. Let's diagram:

"The revised tax code does not apply at all to buildings built before 1900"

P1: BBB1900 ==> not RTC
RTC ==> not BBB1900

"only the first section of the revised code applies to buildings built between 1900 and 1920"

P2: ABB1900-1920 ==> 1SRC
not 1SRC ==> not ABB1900-1920

"since it was built in 1873"

P3: BBB1900

"so the revised code does not apply to the Norton Building,"

C: not RTC

Here we have a valid positive argument invoking P1: BBB1900 ==> not RTC. But P2 cannot be invoked because the Norton Building was not built between 1900-1920, it was built in 1873. Again, we need both premises to be sufficient for the conclusion. Meaning, answer choice (B) does not parallel the method of reasoning in the stimulus.

(C) is not correct because it does not parallel the reasoning displayed in the argument. Let's diagram:

"All property on Overton Road will be reassessed for tax purposes by the end of the year"

P1: POR ==> RTPEY
not RTPEY ==> not POR

"the Elnor Company headquarters is on Overton Road"

P2: POR

"so Elnor's property taxes will be higher next year."

C: PTHNY

This is not necessarily a valid argument. We know that if a property is on Overton Road, it will be reassessed for tax purposes by the end of the year. Just because it is going to be reassessed does not mean that it's property taxes will be necessarily higher. Thus, answer choice (C) does not parallel the method of reasoning in the stimulus.

(E) is not correct because it does not parallel the reasoning displayed in the argument. Let's diagram:

"Since according to recent statute, a building that is exempt from property taxes is charged for city water at a special rate,"

P1: BEPT ==> C@SR
not C@SR ==> not BEPT

"and hospitals are exempt from property taxes,"

P2: H ==> BEPT
not BEPT ==> not H

"Founder's Hospital will be charged for city water at the special rate."

C: C@SR

Answer choice (E) is a valid positive transitive argument: H ==> BEPT ==> C@SR. Thus: H ==> C@SR. Thus, we need both premises to derive the conclusion. Meaning, answer choice (E) does not parallel the method of reasoning in the stimulus.

Therefore, answer choice (C), as with (A) and (E) do not parallel the stimulus. Take a look at answer choice (D). You'll see that either one of its premises by itself is sufficient for the conclusion.

Hope that helps! Please let us know if you have any other questions.

Batman on January 19, 2014

I really appreciate your arduous trial to answer my request!!!!
Thank you so much. You are superb!!!

jamesio on May 4, 2018

Such an excellent breakdown!!